0% found this document useful (0 votes)
276 views19 pages

Reinforced Concrete 1

The document discusses reinforced concrete design, focusing on calculating cracking moments, moment capacity, and load-bearing capacities for various beam configurations. It includes specific problems involving tubular sections, T beams, and precast joists, detailing the necessary calculations and parameters such as stress and moment of inertia. The document provides a comprehensive overview of structural analysis and design principles for reinforced concrete elements.

Uploaded by

Xzk Mallabo
Copyright
© © All Rights Reserved
We take content rights seriously. If you suspect this is your content, claim it here.
Available Formats
Download as PDF, TXT or read online on Scribd
0% found this document useful (0 votes)
276 views19 pages

Reinforced Concrete 1

The document discusses reinforced concrete design, focusing on calculating cracking moments, moment capacity, and load-bearing capacities for various beam configurations. It includes specific problems involving tubular sections, T beams, and precast joists, detailing the necessary calculations and parameters such as stress and moment of inertia. The document provides a comprehensive overview of structural analysis and design principles for reinforced concrete elements.

Uploaded by

Xzk Mallabo
Copyright
© © All Rights Reserved
We take content rights seriously. If you suspect this is your content, claim it here.
Available Formats
Download as PDF, TXT or read online on Scribd
You are on page 1/ 19

REINFORCED CONCRETE 1 Section Area (mm2) Ybot lo (mm") Ad2 [d (y + y(7) :

1 lo + Ad
:

1 75 008 , 75 140 63 +104 .


1102 98 184 .
+
Ig
=
3146 63 18 "mm .
+
+

2 75 (3146 63 10 %
Working Stress Design
+

2 70 000 325 714 58x18" 1160 104 Mar


.
.

+ =
.

196 17
,
.

Problem 1: 3 22 500 ,
200 28 13 183.
+
0 313 18" .
+
Mcr =
44 89 kNom
.

A decorative beam with a tubular section is simply supported on a span of 4.5 m. The tubular section
has an outside diameter of 600 mm and an inside diameter of 300 mm. The allowable cracking stress
=
883 34 10" .
+ =
2263 29 404 .

is 3.2 MPa.
a. Determine the cracking moment b. How much additional uniform load can the joist carry if ' the cracking moment is 50 kN-m?
wL2
Soln : 300mm M =
Wi =
Wswt Wadd Wadd =
11 11-4 62 .

8
.

P
W(6)2
50 Wsw UcAc 24(0 1675) Wadd 7 89kN/m
=
= = =

V 8 . .

N
A
. .

Wi =
11 11
.
kN/m Wsw 4 02 kN/m =
.

V V

. 5m
4
.
Yt c. If the joist is reinforced with 3-25 mm diameter bars with their centroid at 435 mm from the top, determine
/
the moment capacity. Use n=8.
600 mm 208
I
ID" / /

= Ig =

64
; fr 3 2 MPa
=
.

200
X
As =
3 +# +
252 :
1473 mm2

fr
McnY +
Ig
=
Ig =

T(6004-T3004 =
5964 12
.
+
10"m X

& /
First Moment Area Theorem (Q)
3 2 Nmm2 (5964 32 10"mmtS
frig
+

X-208 d 435 =
mm

Mcr Q1 Qu
. .

= .
NA =

300 mm
. .

/ / (

200(x)(*) + 2(b)(x 200)(x 200)(x -200)


d
x 100 X
-

Mcr 63 62 kN-m
-

= -
-

. 1 -

nAs

b. In addition to the weight, determine the concentrated load at midspan that the beam can support.
1 /

Ga
=
8(1473)(435 x) - -
nAs(d x) -

3-15mm & 175 14mm < 200mm Wrong Assumption !


=
=
X .

P
DL SW =
.
.

:
SW 8c Ac = z Calc the .
new value of X J =

v v

Oc 23 5 kN/m2 M= Q1 Qu In I
= =
=
.

Ac :# (Do -
D, 2)
= 1600 2 -

3004 200(x)(*) =
8(1473)(435 x) -

ts MId
.

Ac 212 857 50 mm2


=
, .
=
0 212 m 2
.

·
M=
F x
=
175 03 .
mm

SW =
23 . 5 (0 212m) .
=
4 98kN/m
.
b
&
4 98(4 5)
2
WL2
M 12 61 kN/m
To be Icn bx nAs(d-x
+
·
.
= · = : =

G S
.

zr(2 Ph 200(175 8313


Ix lo + Ad 8(1473)(435 175 03)2
<
M h
.

=
t /
= =
+ -

4 3
.

hdl
2
Ph
Mor =
+ [cr 1153 89 = +
10 "mm4
4
.

63 62kN-m.
=
12 61.
kN +
P(45 fc = 0 .
45fc
=
0 45 (27 5)
. .
:
12 38 MPa
.

P 45 34 kN
=
.

3
f :

M
c. If the 300 mm hollow core were replaced by 300 mm square section, calculate the cracking moment. 12 38 .

= MII75O
3001300
T1(60014
Ig 5686 73 10"mmt Ma 81 62 kNom
-
= = + =

64
.

300 .

3 215686 73 +
184)
Man Problem 3:
. .

300 I
=

300
Consider the T beam shown. The beam is simply supported at a span of 6m and with 2m overhang. The
Y =
300
beam carries a dead load of 15 kN/m excluding the weight of the beam and a uniform live load of 10 kN/m.
Mcr =
60 66 kNom
.

& Compressive strength of concrete is 28 MPa and yield strength of 420 MPa. Calculate the following:
600 a. Cracking moment
b. Critical moment of inertia
c. Compressive stress on concrete
Problem 2: d. Tensile stress on steel
Precast joist with typical section as shown are cast using a concrete strength f’c=27.5 MPa. The resulting e. Capacity ratio of concrete under compressive stress
tensile strength, ft=2.75 MPa. The joist are to be simply supported on a span of 6m. Unit weigth of f. Section curvature at cracking
concrete is 24 kN/m^3.
a. Find the cracking moment 800mm
·
Preliminaries
/ /

80 mm

200mm 158mm Wr
150mm , , , /

Given: .

/
6 Im
t'c 175 L 6m
m
:
MPa = 1 1

81 52 kN 163 04 kN
200 300mm 560
.
.

mm mm

② / f += 2 75 .
MPa Uc 24 kN/m
:
8) 1 52 kN
.

41 13
/ 6-31mm &
.

/
150 mm
3 32m
.

100mm
/
+
t
1 +
③ 325 mm
2 68
. m
/

180mm ,
200 mm Ye 196 27 mm

:

75mm 150 350mm


.

mm
101 19
109 24 kN
.

m
·

500mm /
/ & ⑨

W= =
S . + DL + L Ma 189 24 kN/m
=
.

Mcr =

frtg y :

Exy :
196 27 m -
.
SIN 24(0 232) 5 57kN/m
=
·
=
.
58 74 .

W=
=
5 57 + 10 + 15
.
=
30 57kN/m.
Problem 4:
Soln Location of Neutral Axis
: ·
A 6 m simply supported beam is reinforced with 3-20 mm Ф tension bars is loaded at third points with 20 kN
dead load and 12 kN live load. Use f’c=28 MPa and fy=420 MPa. Use WSD, determine the following:
a) Cracking Moment (Uncracked Stage) See Area Y bot
a. Instantaneous deflection due to dead load.
b. Instantaneous deflection due to dead load and live load.
Mcr =

fr ① 64 000 , 520 c. Initial deflection due to full live load


d. Long term deflection, assuming 50% of live load is continuously applied for two years.
800mm ② 148 ,
000 240 P P 250mm
/ / / /

① Total
80 mm
232 000 , 2m 2m 2m
V
/
242 76 um 358
.

A + Y += A , y + Az Ya
,
1111/
111 400
*
/ / 3 20
-

(
N A .

., 132 000 y + 64 ,
=
, 000(520) + 168 000(240) ,
/

/
2m 2m 2m
V
② 488mm
Y += 317 24 mm um
.

317 24
- 111

Y2
24
=
.

(EI) M =
Px P(x - -

2) -
P(x 4) -

At x
=
0 y
=
0 C =
0
I /
350mm
(E1)0 :

PX2 <x-272-(x- 472 +C Atx 6 =

y
=
0 C = -

44

Moment of Inertia ·

Cracking Moment (570-p3Px-23x-4x Atx 3 :

y
:

-23P23()
Ina Ig + Ad =
fr =
0 611 fo.

Sec Ig Ad
?
fr =
0 62(1) 28
. 9) Deflection due to dead load (4 20kN):

① 34 13 10" .
+
2631 14-18" .
fr =
3 28 MPa. y= 2 Mcn :

Eng Ig =

bh3 2501400-1333 .
33 10 mm +

3 28(6893 16 +
104)
② 3125 6:10" 1001 29-18" Mcn Ma P(3) P(3-2) fr
.

0 62 Xi
.
= - =
=

317 24
.
.
.

Total 3259 73 +104 3653 43-18" Mcr 71 27 kN =


.
-

m Ma =
20 (3) 20(1) -

fr =
0 62(1) 28 Mcr =
3 28. (1333 33 10 .
+

200
. . .

IND =
6893 16 +10 "mm 4 .
Ma 40 =
kN ·

m
fr :
3 28 MPa
.
Mor 21 87 kNim <Ma
=
.
... cracked section !

Analyze as Cracked Section

b) Critical Moment of Inertia (Cracked Section 250mm fo


, I &

800mm to X X
E = 4700 +c 470018 =

N A
. .

/ / / /

e
358
① 80 mm Ec =
24870 06 MPa .

358 -

x
/ /
nAs
X
1
Es =
200 000 MPa
t
,

② 80

E
X
-

nAs 7540 mm : 2
n : :
8 04
350mm
.

/ &

430 -
X

nAs 24
③ / Q1 Q 250(118 + 7540(350-118 24)
.

/
I
=
=

I
.

250(x)(*) =
7540(350 x) -

[cr :
542 75 .
+
10"mmY
350mm
X
=
118 24 .
mm

n= =200000 8 .
=
.
04- For = Ie Ig
NOTE !

As - x322 4825 49mm


= 6 =
.
Ie =

(Man) Fg - (Man) For +


It < Ig : Use Ig

First Moment of Area In =


(2187 (1333 104 -(287(54275 .
33 +
+ IeL Ic :
Use Ich

Q1 QU =
Ie =
671 96 "10"mmt
.

000(80)(x 40) + 350(x - -

80)(x =80) =
8(4825 49)(430 x) -

23(20kN)
:
.

=
x1000 =
9 18 mm
y .

X
=
172 23 .
mm 3(24870 06m)(471 94 104mm4)(1000 3)
. .
=
-

INA Ig =
+ Ad

b) Deflection due to DL and LL


See Area Ig Adz .

① 64 000 ,
34 13 .
+
104 1119 03 x18"
. Icr INA =
Ma 32 (3) 32 (1) 64
= - =
kNom Ecle 24870 06(574 3 184)(10003)
=
. .
+

② 32 280 5, . 22 88 18" .
+
68 65 184
.
+
Icr 3889 74x10"mm
=
. Ie :

(2187) (1333 33-104 [1-(28(542750 .


+ Ecle 14 282 :
,
.
88 kN ·
m
>

③ 38 683 92
,
. 2565 05 104 .
-
Ie =
574 30 10 "mm4
.
+

y z(14
:

,
22332Nm2 * 1000 :
1718m

Total 57 01 .
+
10" =
3752 73x184 .
c) Deflection due to LL d) .

Long term deflection


Ma 12 (3) 12(1) 24
= - :
kNom P 20 + 0 50 (12)
=
.
=
26kN

c) Compressive Stress (tc) e) Capacity Ratio for Compression Yu Ya =

+1
-

Yo Ma 26 (3) 26(3-1) 52 kN
= -
= ·

( (2187](542 75
109 24 10" (172 23) 4 94x100
= (2.87) (1333
Max +

17 64 % 104) +
.

fo Ie
.

17 18-9 18
.

100 33
=
: -
CR + : .
= = + -

28
.

For 3809 74 104


. .
.
+ .

Cfc :
4 94 MPa. Capacity Ratio for Tension Yu = 8 mm Ie =
601 56 10"mm +
.
+

1
58 9
d) Tensile Stress Steel (fs) Ecle 14940 83
2
C R= 100 100 14 82 % kN
.
:
x x
on : m
-

420
. .

23(26)
fs :
Ma(d -x
I cr
f) Section Curvature at Cracking y
:

3(14940 83) .
x1000 =
13 31 .
mm

to
23) 71 27 10"
Dan
109 24 103(430 173
.
+
-

.
.
+

3809 74 +104
. 470028(6893 .
16 +
104

fs =
58 9 MPa .
6 =
0 000000416
.
Mode 3-4
a) Location of Neutral Axis b) Stress on Concrete
*

1
Y
1 4
Q1 QU =
1 :

by + nAs(d -x)

200(x)(*) 200 (212 51)3


.

52 0 8(1963 58)(500 x) I + 8(1963 50)(500 212 51)2


.
= - = -

3
.
.
.
.

2 =
1 658
.

X
=
212 51 .
mm 1 1938 08 10 "mm +
:
.
+

(212 51)
It's
E Mx 120 104
+

20 : Y + : Yo+ Y fc
.

p 0-walang
:
midspana
: : .

1 + 509 I 1938 08 +104


.

1 658
Mo 658(13 32) Y+ 22 : 13 16
.

1 08 + 13 32 MPa
658y
: =

Yo
:
1 = f

1 + 50(0)
. . .
. . .

Xy =
1 658
.
Yo 22 08
:

.
mm Y += 35 40 .
mm c) Stress on Steel

M(d 120 101 (500 212


-x) 51
ts
+ -

.
= .

Problem 5: 1938 08 104 .


+

A reinforced concrete beam has a width of 280 mm and effective depth of 520 mm. It is reinforced for
fs :
17 80 MPa
tension only with 5-28 mm Ф bars. f’c=21 MPa and fs=128 MPa. Assume n=9. Use WSD.
.

a. Determine the location of the neutral axis


b. Calculate the compressive force on the concrete. Problem 7:
c. Determine the moment capacity of the beam. Design a section of a concrete beam reinforced for tension only. The beam is simply supported on a span of 7.5 m and
carries a total load of 18 kN/m. Use a minimum width of 250 mm. Use 20 mmФ with yield strength of 280 MPa. Use
f’c=21 MPa an use modular ratio n=9. Sketch the section of the beam.
280 288
/

X
/ /

X
:
I tex
1
250
/
250
/ /
fo
43
= 28 N A. X
I /c :
Efexb
X
.

/ ,

d
d -x 520 -

x
/ ,
N A.
.

d -

5
h
I / ↓ > T
=

Asts d -x 500 -
x

As nAs
I
I / /T Asts
I
=

As nAs
/

a) Location of Neutral Axis b)


.

Compressive Force

Q1 Q = C :
Etoxb Procedure :

280(x)(*) 9(5 +y 282)(520 = x -

x) It a) Allowable Stresses fa

X
=
136 77 .
mm
fo
234 77 .
:

528 236 77
128/9
-

.
fo 0 .
45f =
0 . 45(21) 9 45 MPa :
.
"He /c :
Efexb
X

fc :
11 89 MPa
.
+s [ 0 50 ty 0 50 (280)
.
=
.
:
140 MPa
N A.
d-
( (11 89) (236 77) (280)
.

C :
.
.
b)
.
Compute the actual moment due to load M
d
WL? 1867 532
-

C 394 13 kN
:
.
M =
·
: 126 56 kN .

/T Asts
I
=

c) Moment Capacity c) Calculate the effective depth of the beam

fo
bx =+cxb(b -)
fs/n
[c = + nAs(d -x) Xd
:
-
X
M

280(236 77)3
9(5x x282)(520 236 57) E(9 45) (2945) (250) (b-d12 65)
45
In =

3
.

+ -

.
9 .
14019
d -
X
126 56 103 .
+ :
.
.

Icr =
3461 63 10 "mmY
.
+ d -

x
=
1 65x
.
d 569 89mm
=
.
=
570 mm

d
X
=
X
:
215 09 mm
2 65
.

·
Due to stress on steel ·
Due to stress on concrete Determine the required reinforcements (A)

Is. Ma(d-x
Ich
fc :
Max
Ic
&F =
0 C T =
# of 20mm & bars

Ma (520-236 77) Ma (236 77)


[fexb #82
128
18- 11 89 10.4 Asts 6-20m s
.

5 78 =
.

: = + =
+ :

9 3461 63x184 3461 65 104


. .
+
.
.

Ma 173 82=
.
kN ·

m Ma :
173 83 kN
.
.
m ! (9 45) (215 09)(250)
. .
=
A , (140)

As :
1814 82 mm
Problem 6:
.

A rectangular reinforced concrete beam having a width of 200 mm and effective depth of 500 mm is
subjected to a service moment of 120 kN-m. The beam is reinforced with four 25 mm Ф bars. Use d) Arrange the reinforcement
modular ratio n=8. Use WSD.
a. Determine the location of the neutral axis.
b. Calculate the stress on the concrete.
~
Stirrup (10mm min . e m

c. Determine the stress on the steel. Concrete Cover


SS
(40mm min )
200 200

tctexb
.

Ss

/ / /
S[25mm or db
I
X
X
250 250
500 N A..

/ ,
Check spacing of bars Check spacing of bars
d -x 500 -
x

250 =

40(2) + 10(2) + 6(20) + 5 250 2 (48) + 2(10) + 4(20) + 35


=

I / ↓ > T
=

Asts
As nAs
I S
=
6 mm 15mm Not ok ! S
=
1 3 33 .
mm 25mm Not ok !

As :
4+F yx152 :
1963 50 mm 2 .
1

Where :

- -
d

h d+ y
=
y 40 + 10 20 +
=
+ E =
82 5 mm
.
C: Etoxb
25 ① 1 n 570 + 82 5 652 5 mm
=
.
=
.
C (2n-1) As
=

Is
Y
T

h 650
=
= Asfs
say mm

/ /

250 Steel
·

As :Asp
,
=
2627 51 mm2 .
·
Check the stress on Compression

Final Design
·
Moment to be carried by compression steel Iffs2fs >
Compression steel will yield

250 mm Ma-My =
200 165 58
-

.
=
34 42kN .
.

m
-
Ma t's fy
/
t
I
148
2-16 mm &
Mc Asets (d-d')
:
to
Xb d
=

d -

xb >2(93 - 85
-
=

515 - 194 43
-
65 :
95 82 MPa
.

34 42 18" As (140) (515-85)


-
.

+ =
.

Asc =
571 76 mm2 . Tz Cs =

650mm 10 mm &
Total tension steel Asets (In-1) As it's :

As Asit Ase
: =
2627 51 + 571 76
.
.
571 76(140) .
=

[2(9) 1)(A's)(98) -

6-10mm4
As = 3199 27 mm? .
As =
884 53 mm2 .

Problem 8:
It is desired to find the steel reinforcements at the most critical section in bending of a beam having a SUPPLEMENTARY PROBLEMS
cross section of 400 mm by 600 mm (b x h) with an effective cover of 85 mm for reinforcements. Problem 1:
F’c= 21 MPa, fs= 140 MPa, n=9. Determine the required area of steel reinforcement at the following A reinforced concrete rectangular beam having a width of 350 mm and a total depth of 600 mm and effective
condition. depth of 530 mm is reinforced with 4 − 28 mm∅ tension bars. Material strength are fy = 420 MPa, f′c = 28
a) Balanced condition MPa, fr = 3.28 MPa and n = 8. Determine the stress in concrete and steel caused by a bending moment M =
b) If the beam carries a bending moment of 150 kN-m. 60 kN − m.
350
c) If the beam carries a bending moment of 220 kN-m. / &

Uncracked
/
400 mm
/
400
/
fo X 9) Check if Cracked or

Xb
c = texb 538
600
/
Ma < Mer-Uncracked fi
:

Mc GOTOu30 .

538 12
X
Ma > Mcr- Cracked
-

NA

515
I
4 28-
fc =
2 86 MPa

28/35
.

/
600mm /
3

Mcnifrig
.

d Xb-

300
Asb nAsb
I / > T =

Asts Mcr 68 88 kN m > Ma


= . :
Uncracked
I
.

85
I

b) Ma =
80kN m ) Mcr : Cracked
·

I, n
=
350(194 39 8(4- 282) (530 194 39
.

+ * -

a) Balanced Condition Moment Capacity at Balanced Condition Q1 QU =


Fir 3076 33 -10"mm +
:

fc 10 45f'c .
=
0 45 (11) 9 45 MPa
.
=
. Mbal Obal =

(d-tXbal) 350x(z) 0(4 # 282)(530 x) =


+ + -

f:x S439-537MPa20 45(28)


-
.
:
12 MT

Etc xyb(b EzXbal) f M(dx) 6 04(530 39) <f 74


194
85 % -

fs 140 MPa 194 39mm I SM


.

= = - = :
X .
. , .

3076 33 -104 .

By ratio and = (9 45) (194 43) (480) (515 194 43)


.

proportion
:

Problem 2:
-

. .

fcrfs/n A reinforced concrete beam having a width of 300 mm and an effective depth of 530 mm is reinforced
Mbal =
165 58 kN -

m
with 5 − 28 mm∅ tension bars. Material strength are fs = 128 MPa, f′c = 27 MPa, and n = 8. Determine
*
.

the following using working strength design:


Area of Steel at Balanced Condition a) Balanced moment capacity of the beam
Xb 5/5 Xb
-

b) Moment capacity of the beam


Xp 194 63
:
mm C T =

+ 10 45tc
.

/ x/C
EfcXbb =

Aspts
Yes Xb
:

(f)(b)(xs) a) Balanced Moment (Ms)

(9 45) (194 63) (400) =


As (140) Balanced Design &M += O
530
MB
. .

, /

As :
2627 51 .
mm
?
d Xy -

to 0 45fc .
Ma =
c
(d =z) -

+(d -)
3
by 200(186 93) T =A
Is Ooty fs = 0 50 fy
t

b) . At Ma =
150 kN ·

m INA =

3
-

nAs(d x)2 -

9 (A , ) =
.

.
Mp =

515 -
186 93 .

2
400x3 200x 300
MaLMp :
SRB +
(515 -x)(515 x)2 As
-

2366 89mm
=
:

3 .

400x3
(515 x) fc 8 45 (27) ) Moment Capacity (Ma)
2
Locate N A IN = + 200x -
: =
12 15 MPa b
3
. .
. . .

ts M(d x)
As # 3078 76 mm2
-

Q1 QU 5 182
Proportion
:
Ratio +
= + =
= =
& .

N INA

#
140150 10" (515 x)
400x(E)
-

nAs(515 x)
+
= -
I QP QU =

9 400x3
+ 200x2(515 x) -

=200 300x( 2 )
z
nAs
X
=
186 93 mm
=
8(3078 76)(530 x) .
-

c ) At
. Ma 200 kN
= ·

m Xb 228 76
=
. mm X
=
124 11 .
mm

Ma) Mb :
DRB
Mo = fc Xb(d -) I =
300 (224 1173
.
3
.

+ 813078 76) .
(530 -

224 11) 2
.
=
3430 20 "10"mm +
.

fo
:

( (12 15) (228 76) (300) (530 228 74) .

Is Mdx) 128M(530-24
. -

is'G taxab
-
.

400 3430 2 +184


.

,d
/

(2n-1)A's I
fishn
2
85
Cs
=

My 189 Ma 178 90 kN
=
>
2 17 kN =
m
.

/ =
m
·

(2n-1) As
·

Xb

,
NA Ma =

T(d -)
d -d' Ma Ma
(538 224 1)
MB
d -d'd
-
-

3078 76 (128)
.
-

, :
.

d Xb-

Ma 179 41 kN m
·
=

nAsb
.

T TiAsifs :TEAsets
I

I
> <
!
Problem 3:
A rectangular concrete section is reinforced with 3-25 mm ϕ bars in tension and has a width of 300 mm
and a total depth of 600 mm and effective depth of 540 mm. Use 𝑓′𝑐 = 27.6 𝑀𝑃𝑎 and 𝑓𝑦 = 414 𝑀𝑃𝑎 and
n=8. Calculate the following:
a) Cracking Moment
b) Moment of inertia of the cracked section
c) Moment capacity of the section
300 fo
I /

a) Cracking Moment
X

S
Mor =

fri
548
600 fr =
0 62 27 6
. .
=
3 26 MPa
.

540 300 (600)3


Ig
X
5400-10"mmt
-

= =

12
3-25mm
3 26(5400 +
104)
Mar 58 68 kN
.

E
= =
m
.

300

Q4 QU Concrete
.)
b =

c) Base on

300x(E) 8(3 7 252)(540 x)


= + + -

fe = 0 45 (27 6) .
.
=
12 41
.

Ma(103
X
=
170 38 . mm tc =

Max 12 .
42 :

2184 11 +04 .

38
300(170 8 (3x 252) (540-170 38.
.

[, = + +
.
Ma 153 =
.
38 kN ·

Icr 2104 =

,
11 "10"mm4 Base on Steel

fs -0 50(414) 207 MPa


:

I :

Mad -x) 187 , :


Ma(540-103
2104 11 +104
.

Na 147 30 =
.
kN ·

Problem 4:
Consider the beam section shown. Material strength is fs = 165 MPa, f′c = 27.6 MPa, and n = 8.
Determine the moment capacity of the beam section.
150 200 150

100

200
Singly Reinforced Beams
Ultimate Stress Design

bu Ec =
0 003
.
0 85 fo
.

/ / / /

fo C 0 85f'c abw
=
.

a = B, C
<
<
C

/
d
h de &
d-EMn
Es
·
I 7 T Asfs =

As Et
/
Strain Stress Whitney's rectangular
Diag ram
Diagram Stress Digram
Analysis of Singly Reinforced Rectangular Beam
300 0 003 0 85tc
Problem 1: &
.

&
.

'a/z
&

A rectangular concrete beam has a width of 300 mm and an


effective depth of 437.5 mm and is reinforced with 4-28 mm ϕ C
d 4/
C 0 =
.
85 %c abw
bars. f’c= 42 MPa and fy = 415 MPa. &

437 5
1. Determine the balance steel reinforcement .

2. Compute the maximum steel reinforcement


d -

9
3. Compute the minimum steel reinforcement As
4. Determine the nominal moment capacity of the beam / 1 >
T Asts =

4- 28mm & fs/E


5. Determine the design moment of the section
6. Determine the maximum allowable live load that the beam fylE -
Ba

could carry if the beam have a simple span of 6m. f'c =


42 MPa

fy =
415 MPa

0 05
a) Ask Balance design RP B: 0 85 . (42-28) 0 7570 45
: =
-

. . .

Cb d Cb
&F
-

0 003
.

0 85%i, .
=
O
0 003
.
I

Eb
a =
B C ,
=
0 .
75(258 (2) .
=
193 97mm
.

& &

: =
al 4/5
4/ = 0 85 f'c Abb
C C Tb :
Eb 200 000
=
0 002075
.

Ob Ab .
,

Cn 437 5 - y 0 85(42) (193 97)(300)


As ofs As
.

0 85fc abb
.

&
.

: = =

d
/ .

0 003
. 0 002075
.
415
d-E Mny
Cb As
,
d ob-
:
258 61 . mm
=
5005 83 mm2 .

Eb
/ 1
To Asofs
:

b) .
Asmax 6) Asmin =

I bod I
85tB (3) , 1300(437 5)2(300(437 5)
0
0 02
Pmax
.
= I
= , .
. .

Asmax =
0 02765 (300) (437
. . 5) Asmin =
512 .
41mm2 > 442 77mm .

Asmax =
3629 06 mm .
: Use Asmin =
512 41mm2
.

.) As 4 x
d :
# +
28 2443mm
:
fs- fy
Es 0 003
AsminEAs Asmax
.

d C -
C

fs/200000 0 003
Using
.

Mechanics 437 5 127 25: 127 25


-

. . .

Assume Tension Steel will yield ! fs =


1462 87 MPa 7
.
fy

fs fy = =
415 MPa : Tension Steel will Yield !

&F O :
fs fy

C
=T Mn =

T(d ta) -

a
= As = 24 =
954 m ! =
2463(415)(437 9544) .
5 -

c =

&- 9544 :
Im s Mn 398 =
. 41 kN .

e) MulQMn

0 .
901

0 65-
.

Ey
Problem 2:
A reinforced concrete beam has a width of 300 mm and total depth of 500 mm. It is reinforced with 6-28 mm ϕ bars. Compressive strength of
concrete is 28 MPa and fy = 420 MPa. Use 10 mm ϕ stirrups.
1. Sketch the arrangement of the tension reinforcement
2. Compute the depth of compression block
3. Compute the stress on the steel
4. Compute the design moment of the section
0 003
& &
.

0 85t's .

a) minimum

d L C
= 0 85f'c ab
C .

d h y
= -
&

&

/ d 418 33
=

Mn
.

S = 15 + /

/ <
Yz S

i
Y As
/ ↓
S > T =
Asts
1404140 200 Es

#x282
/ &
300 As :
6 + :
3695mm

Let Azg : b) Assume As will yield (ts fy)


:

Recompute a
:

y 40 + = 10 +
28 =
64mm =T
C f # fy

Yc 40 + 10 + 28 + 25 +
=
2 =
117mm 0 .
85(28)(a)(300) 3695(420) =
C T =

jA+= A , y + AzYz
, a
=
217 35 . mm 0 85 % 'c ab Asts
.
=

(6) =
4(64) + 2(117) c=
& 23 255 71mm
=
. a
=
Bc ,
=
0 85c
.

j
=
81 67.
mm Check if As will
actually yield ! + :
600
(d =c)
(d) 36957600(418 . -c))
33
d 500 81 67 418 33 mm
= -

.
=

. + 5
=
600 Actual Stress 0 85 (28)
.
(0 85c)(300)
.
=

to =
600
(418 33255H
.

c =
248 83 . mm

fs 381 57 MPa < 420 MPa 211 51


=
a
:
.
.
mm

... Will not yield !

3) + =

600 (dc) d) Mn
=

600(418 33-208 .

fa < ty -6 =
0 65
.

fs =
408 71 MPa
. PMn =

PT(d E) -

T
Asts
=

& Mn 0 =
.
65/3695(408 71)](418 . .
33 -

21151)
& Mn =
306 83 kN .
·

m
Soln :

a) B ,
=
0 85 . .)
b Moment Capacity Reduction Factor c) Design Strength
P :

As :187 5)
=
0 01.
c
=9 1258155
.

.
98m $Mn PAsty (a-)
=

7)(375 13258)
PatMatt+((
Es =
0 003
.
(ac) 003(3755g
=
0 .
=
0 833
. (1875)(414 .
-

2 , 0 0042/
=
.

& Mn =
200kN .

Pbal =
0 0284
.
Ex
=

+ =4470 :
0 00
.

P Pal :
Tension steel will yield ! (fs fy)
:

0 =
0 45 + 0
.
25)0.)
.

Apply Equilibrium Condition =


0 65 + 0
. .

25) .00-0000
C T =
0 =
0 833 .

0 .
85fc ab Asts
=

1875(414 7)
=
.

a =

b 0 85 (27.
.
6)(250)
a
=
132 58 .
mm

a) Actual Tensile Stress +s :

600(dc) . ) Compressive Force


b

B: 0 .
85 =

600(410-24T C 0 85 fi ab
:
.

p. As34A10 = 0 03.

fs =
393 98
. mm
= 0 85 (27 1) (210
. .
. 37)(300)

85 R (d)
0
Phal C 1453 74 kN
.

= , =
.

85)(404
1
85
0
(0
.
.

=
.

Phal =
0 0279 .
c) Factored Moment Capacity
P Obal -: Tension will not yield ! (forfy) 0 :
0 65.

Apply Equilibrium Condition pMn =

Pc(d ) -

C T =
a
=

B ,
c +
0 85c
.

pMn =
0 .
65(1453 .
74)(410 210.34)
-

0 85tc ab
.
=
Acts + =
=

600(dc) ↑ Mn :
288 03 kN .
-

0 . 85(27 1)(0 85c)(300) . .


=

3690(600(410 c))
c
=
247 49mm .

a
=
210 37 mm .
Step 1 :
Check if tension will yield As =
4 + +
202
-
2
=
1256 64mm
.

B ,
=
0 85.

P. - As 1254 =
O
d 370 +
=
40 =
400mm

85 (d)
0
Ph R d+
.

= , =
370 + 68 =
430mm

85(28) (0 85)(420)(1)
0 .

=
.

Phal = 0 0283
.

PLPbal · · Tension will yield ! (ts fy)


=

Step 2 : Det the .


depth of compression block
C T =

0 .
85fcab =

Asfy
0 .
85(28)(a)(250) =
1256 .
64(428)
a =
88 70 . mm

c
= =
104. 36 mm

Step 3 :
Det the nominal
.
strength

Mn =

T(d 2) -

=
1256 .

64(420)(400 8878) -
.

Mn 187 71 =
.
kN ·

Step 4 Det the


:
.
design strength

f =
600(d = ) +

=
600 (430
fs :
1872 21 MPa > 1000 MPa
.

6 =
0 90.

$ Mn =
0 90 .
(187 71) .

↓ Mn = 168 94 kN .
·

m
Design of Singly Reinforced Rectangular Beam
A beam is simply supported on a span of 5 m and carries the following loads:
Super-imposed dead load = 16 kN/m
Live load = 14 kN/m

Properties:
b=300 mm h=450 mm
d=380 mm f’c=30 MPa
fy=415 Mpa S.G.=2.4

1. What is the maximum ultimate moment at ultimate condition?


2. What is the required depth of compression block?
3. Find the required number of 16 mm ϕ tension bars
0 003
& &
.

0 85t's .

LL
C
d L = 0 85f'c ab
C .
V v

&
DL
& v V
380
I/
Mn
11 IIII

U =
1 2 DL + 1 6th
.
.

<

As
S > T =
Asts Wu
70 Es
/ V v
300 111 11I

a) Mu ? =
b) . a
=
? C T =

DL S W + 16 kN/m
=

.
.

Design the beam such that it will fail in tension ! 0 85 (30) (a) (300)
.
=
1084(415)
SI
. .
=
VcAc SGcVeAc =
① fs #fy >
Ductile failure a
=
58 81 . mm

S(
. .
=
2 4 (9
.
. 81) (0 3 .
+
0
.
45) =
3 .
18 kN/m ② =
0 90.
> Tension-controlled section +c 30 MPa = -
B ,: 0 836 .

Wu = 1 2 (3 18 +
. .
16) + 1 6(14).
=
45 .
42kN/m Solve for "a" C
:

& =
70 35
.
m

42(5) Mu 141 94 184


Wul245
:

Mu 141 94 kNim Rn 3 641 In terms


.
=

of
.

:
stress
=

= bd2
=

9(300) (380)"
= .

0
.

g .

p :
0 .

85 /1-1-2) =
0 009509
.
fs 1000 MP a

As P bd =
=
0 .
009509(300) (380) =
1084 03 mm .
270 005 .

+ =
600
(d ) =
2640 94 MPa .

: Tension-Controlled Section

a
=
58 81mm
.

c) # of 16 & bars Alternate Solution :


mm

Nin
:08 C T c=
q 70 33mm
:

5 39
= =
. .

Nic 6-16 =
mm &
Mn =

T(d ) -
f =
2641 86 MP a
.

Mn =
c (d 4) -

C T =

& Mn 60 85f'c ab (d-)


=
.
0 .
85(30)(58 80)(300) A , (415) .
=

& Mn ? Nu As 1083 =
.
9 mm

↓ Mn ? Mu
Mn =
141 94 10" 0 90
.
+ =
.
[0 85(30)(a)(500)(380 - a ))
.

a
=
58 80. mm
A rectangular beam having a width of 300 mm is reinforced for tension only. The beam carries a factored moment of 540 kN-m.
Concrete strength if f’c=28 MPa and steel yield strength fy=280 MPa. Design the beam under balanced condition.
a.) Determine the required steel ratio
b.) Determine the required effective depth of the beam
c.) Determine the required area of steel

Soln :

Ec =
0 003
.
a) Steel Ratio at Balanced Condition (Pb)
Ob = 0 85 .

Tc s (400) d
, : p =
0 45. (Balanced Desig
a
=
0 .
8528 (0 .
85)(280)(1)
d -

Cb
9 =
0 04926
.

Es :
fy/E

b) Reg
· effective depth (d) c) Reg
. As

Mu =
Pfobd"w(l-0 59a) .
As bbd = =
0 . 04926(300)(532)
Wa
=

P (t) 04924(280) =
0 .
=
0 42
. As = 7862mm ?

540 +
104 =
0 .
45(28)(300) (d) 2(0 . 4926)[1 -

0 .
59(0 .
4924))
d 532mm
=

A 6m simply supported beam with 2 m overhang carries an ultimate load of 45 kN/m. Use f’c=35 MPa and fy=400 MPa. Tension
reinforcement shall be placed at 65 mm from the extreme tension fiber.
a.) Determine the dimension of the beam if h=2b.
b.) Calculate the required depth of compression block.
c.) Calculate the required area of steel reinforcement.
d.) Sketch the cross section of the beam if the bar diameter is 20 mm.

Soln :
45kN/m

v - v V
9) Dimension of the beam if h 2b=
03 =

0 .
00590 .
003
0 003
6m
· : d
.

↑ c
120kN &
N a
=
B , c C 0 =
85% cab
88(d)
C
.

128
a =
3 ,
c =
0 .
=
0 3d.

98
d 45
d
d- Mr h = d + 65 ;
b
+=

8/3 Mu =
PMn =

PC (b -) -

<
T =

Asty
900 85(35)(0 3d)(4 +45))(a -)
160 E
160 104 = =
0 .
. .

-
150
E + 0 003 .

d 340 =
13 mm
Let 2 0 005 =
.
.

0 05 340 13 + 45
. (35-28) b
.

& 0 85 0 80 202 57
=

225
= = =
=
-

, mm
2
. .
.

-
98
h =
d + 65 =
340 13 + 65
.
=
485 13 .
=
425mm

b h 125mm
+ = =
425 mm

b).
Depth of compression block (a) c) Reg . As d) Detailed Section

600 (d )
1394
Mu =
Pc (d - ) f = =
1470 75 MPa) 1000 MPa
. N2o :

#120
=
4 44.
= 5-20 mm &

160 + 10" =
0 .

900 85(35)(a)(225))(340 2)
.
-

C T
=

a
=
83 45 mm .
. 85 (35) (83
0 . 45)(225) As (400) =

425

c =
104 31 .
mm As :
1396 48 mm2 .

225
A rectangular reinforced concrete beam having a simple span of 4.5 m carries a service dead load of 20 kN/m and a service live
load of 35 kN/m. Steel covering is 75 mm measured from the centroid of the tension steel. Concrete strengths is f’c=28 MPa and
steel yield strength is fy=420.
1. Calculate the expected design moment of the beam
2. Which of the following gives the least dimension (bxh) of the beam using the maximum allowable reinforcement ratio?
a. 200 mm x 450 mm
b. 200 mm x 475 mm
c. 225 mm x 450 mm
d. 250 mm x 500 mm
3. Determine the required steel area using the dimensions obtained above.

a) Wu =
1 2 DL + 1 6LL
.
.
=

80 kN/m

Mu =
Wh =
201 5 kN m .

Mu
b ) Ductile Failure bd2
.

= +ow (1 -

0 . 59w)
202 50 +
104
fs fy
.

0 90(28) (0 271) [1-0


. . . 59(0 271)]
.

Tension Controlled -
bd = =
35 , 295 475 87mm3
, .

Mu =
Pfc b d z(l-0 592) >
.
bd2

w
=

p a) 200mm 450
*
mm 18 , 125 000 ,
X

85 (3)
0
Pmax Po 005 .
b ) 200mm 475mm 32 000 000
.

= =
= X
,
,
.

Pmax = 0 01806
.
.
c ) 125 mm + 450mm 31 440 625
, ,
X

w
=
0 2709
.
.)
d 250 mm
x 500mm 45 156 250
, ,

ei) 300mm *
450 mm 42 187 500
, ,

c) Mu =
Of c bd w(l-0 59w) .

x = 0.)
201 5 104 0 9(28) (250)(425)
.
+ =
.

2(2)(1 -

0 .
592)
1 493
p wi 0 099
:
W .
· =
=
.

w 0 202 P- why 0 01347


=
- =
. .

As Pbd = =
0 01347 (250)
.
(425)
As = 1431 19 mm ? .
Doubly Reinforced Rectangular Beam
A reinforced concrete beam has a width of 300 mm and an effective depth of 600 mm. Compressive reinforcement if required will
be placed an effective depth of 60 mm. F’c=27.5 MPa and fy=415 MPa. The beam is to support a factored moment of 670 kN-m.
a) Determine the area of tension steel reinforcement if the minimum strain in the extreme tension steel is permitted to be 0.004.
b) Determine the area of tension steel required if the minimum strain in the extreme tension steel is limited to 0.005.
c) Determine the number of 28 mm ϕ bars required as tension reinforcement if reduction factor is 0.90.
d) If 2-28 mm ϕ steel will be added, determine the maximum tension steel area required for tension-controlled section.

300 0 003
.
0 85t'
.

70 Es
2
Cs =

Asts
As db -
C =
0 85% cab
C
.

600
d-d'
d-E Mu
,
600 -

As

Es Max SRB
2 >
Tz Asefy =

TF As , fy
As , Asmax
"

a) As for Ex =
0 004
. Asi Po oopbd .
Mu = fibdz[1-0 59w] .

Muc GT (d d')
= -

85(5)(0 85)(3) [0004-0002075)


600
.os (Asz)(4(4)(600 40)
c

00
-

p 0 65 + 0 25 104 0 81
.

40 63
=
I =
0 . . = = -

0 004 004
.

0
. .
.

.
.

C =
157 14 . mm Po 0r4 .
=
0 02052 .
6 =
08 .

Asc =
334 82 mm.

a
=
218 57mm .
W
=

00 .
004 0252(c) =
0 .
Mu = 0 . 81(27 5)(300(600) (0 31)
. .

/1-0 59(0 31))


. . A , 3693
=
.
6 + 334 82 .

-O a
w
=
03
.
Mu ,
=
609 37 kN .
·

m As 4029
= mm2

As =
0 .
02052(600 300 + Mu ,
< Mu 670kN = ·

m
:
DRB

Compression steel will yield ! 3693 6 mm2 Muz 670-609 37 60 63 kN


=
:
As ,
=
m
·
= . .

.)
b As for 21 :
0 005
.
As , 90 005 .
Mu =
Pfibd2w[l -
0 .

59w) 63 24 104 0 9 (Asa) (415)


.
+ =
. (540)
(5) (0 85)( (27 5) (300) (400)(0 27)/1- 0 59(0 27))
C 600 -

0 003
.
=

0 005 .
00 005 .
=
0 85.
.
=
0 90 . . . . . Ase :
314mm

225
c =
mm
00 005 0 018 Mu , 606 74 kN m
As 3240 + 314
-
= :
. .

a
=
191 25 . mm z
=
0 018 .

() =
0 Mu ,
< Mu 650kN =
·

m
:
DRB
As =
3554 mm2

-Ona
As =
0 . 018(300) (400) Muz =
670 -
686 74 .
=
63 16 kNm .

As , 3240mm =

:
Compression stul will yield !

PMn =
Mu

c) # of 18mm & for 4 0 90


=
.

Nes =5 =
5 77
.
=
4px

.)
d A =
2-28 mm Muc PCs (d-d) =
Mu . =
&T .
(d 3) -

As =
2 +

# +
282 = 1232mm
Muz =
0 90
. (1232) (415) (600 40) -
421 52 18" 0 9 (Asc)(415)
.
+ =
.

(600 191 25)


-
.

For tension-controlled E+= 0 005 and .


0 =
0 90
.

Muz =
248 48 kN
.
·

m As :
2238mm

·600 c
=
225m
=
191 15
Mu ,
=
670-248 48 .
=
421 52 kN
.
·

m As As =

,
+ Ase =
1232 + 2238
a . mm

fi =
440Mia :
Cs yields ! (As
=

Ase) As =
3470 mm
A RC beam section is 375 mm wide and 500 mm deep must resist a dead load moment of 105 kN-m and live load
moment of 210 kN-m. F’c=21 MPa and fy=415 MPa and effective concrete cover of 65 mm.
a) Determine the required nominal flexural strength of the section.
b) Determine the maximum steel ratio allowed for tension-controlled section.
c) Find the number of 28 mm ϕ tension steel.
d) Calculate the stress in compression steel.
e) Find the number of 28 mm ϕ compression steel.
375 0 003
.

& / S /
65 Es
I

Cc 0 85 fab
Cs Ast's:

As
=
d > .

500
Mnmax Mnz
L

As
Ti :

Asimaxty Tz Asefy
65
=

/ S
Es =
0 005
.

Step 1 :
Solve for Mu

Mu =
1 2 DL + 1 62L
.
.
=
1 2 (105) + 1
. .
6 (218) =
462kN ·

Step 2 :
Check if DRB/SRB

Mnmax SRBL Mn Pmaxsrb Po =

.
005
=
0 .

85 (3) Mnmax =
T, (d ) -

... DRB 00 . 005


=
0 .
85(45) (0 85)(8) .
=
0 01371
.
=
2234 .
44(415)(435 13844) - +
18-h

GMn ? Mu Asmax 00 005 =

.
bd =
0 01371
. (375) (435) =
2236 44mm
.

Mumax =
339 39 kN
.
.

m < Mn 513
=
.
33 kN ·

& Mn =
Mu Cc , =
T
..
Design as DRB

85(3 435)
=Po zd
0 0 85(21) (a)(375) 2236
.
=
.
44(415) c =
; a
=
Bc ,
= 0 .
+ =
138 66mm
.

0 9 .
a
=
138 66 .
mm Mn =
fibd2 z(1-0 592) .

Mn = 513 33 kN m . w =

p 80 . 00s

Step 3: Solve for As & As

Solve for As As =
2236 44 + 1133 18 .
.
Cs Tz=

As As :

,
+ As As =
3369 42 mm2 .
Ast's Asify
=

A smax for SRB Nes =

336962 =
5 47.
ps
= As (340 92) .
=
1133 . 18 (415)

Mnc Mn-Mnmax =
Solve for As As =
1301 97 mm2
.

1302 97
513 33 339 33 Check As will yield
.

Nes 2 12
=
3 pas
-
:
= =

#
. . .

+ 282

Mnc = 174 kN m +i =

600 (c[a)
Mnc Tz(d-d') = =

600) C45T
174 184 Asz (415) (435 45) t's 360 92 MPa
- =
+ = .

Ase = 1133 18 mm? . fs < fy :


CSINNY !
A reinforced concrete beam has a maximum effective depth of 630 mm and is subjected to a total factored moment of
1062 kN-m. Effective concrete cover for both tension and compression steel is 70 mm. Use f’c=28 MPa and fy 414 MPa.
Determine the required number of tension and compression bars using a bar diameter of 25 mm. Sketch the bar
arrangement.

350 0 003
.
0 85t'
.

70 Es
2
Cs =

Asts
As db -
C =
0 85% cab
C
.

638
d-d'
d-E Mu
,
630 -

As

Es Max SRB
2 >
Tz Asefy
=

TF As , fy
As , Asmax
"

Soln :

moment
Step D Det the
design Step 8 Calc the As
: :
. .

reg. . area

Mu = Mn As As =

,
+ Asa : As As max,
=
for SRB

Mu 1062 =
Muz Mu-Mu =

,
=
1062-797 38 .
=
264 62 kN.
-

Step2
:
Check if SRB or DRB Muc =
&Tz(d d') -

Consider the strength capacity at SRB condition . 264 42 .


+
184 =
0 . 9(Asc)(414)(638 5) -

& Mnmax at SRB < Mu As :


1269mm2

Design as DRB 1: 4041 + 1269 =


5310 mm
2

Calc the . max.


capacity at SRB. Step & :
Calc the reg. . .
As area

Let Mu =P Mnmax at SRB Check if compression steel yields (tsty)


For ductile design ,
consider Ex 0 =
. 005
t :
600
(136.28-70) =
422 24 MPaCSNY
.

% 003.
= 0 .
85(44) 85)(j) (0 .
=
0 01832 .
Cs T :

Is 10 005 bd (350) (430) Asty Asefy


2

,
= =
0 01832
.
=
4041 mm :

As :
As max for SRB As Ass 1269
: =
mm2
,

Mu ,
=

PT (d-E) d .
=
0 90
.

Step O
:
Dut the .

reg .
no .

of 25 mm &
For Asi
T

Cc , :

0 85 (28)
.
(a)(350) 4041(414) =
N25 :30 22
=
10 81
. = 11 pas

bar
a
=
200 84 . mm Due to beam dimensions ,
arrangement is not possible .
Try 36 mm
c
=
236 18 . mm Nzu =

+30342 =
5 22.
= 6 pa

Mu ,
=
0 . 90(4041) (414) (630 -

20084) .

For As:
3 25-

Mu ,
:
797 38 kN .
·

m N2z :4 2
=
2 59
.
=
3p

Mu . LMn =
1062 kNm

4 34-

DRB
..
Design as
A rectangular beam has a width of 300 mm and effective depth to the centroid of the tension reinforcement of 600 mm. The
tension reinforcement consist of 6-32 mm ϕ bars is placed in two rows. Compression reinforcement consisting of 2-25 mm ϕ
bars is placed 65 mm from the compression face of the beam. Use f’c=35 MPa and fy=414 MPa. Calculate the design moment
capacity of the beam.

300 0 003
.
0 85t'
.

I
Es
I 2
2 =

Asts
As db -
C =
0 85% cab
C
.

c -
d

d
de I t
d-d'
Mm
d-E Mu ,
,
As 625 -

C
Es
I
TF As fy
>
Tz Asefy
=

Et Max SRB
As , Asmax
#
"

As 322 > 4825mm2


:
x + =

As # : +
252 2 + =
982mm

Assume steel yields .


Step O compression Step ⑤ Calc the moment capacity
:

design
.
:
.

Is ty :
As As , + Asa
=

-
[FH =0 ; Cs =
Tz As =
4825-982
d
dy =
600 + 2 =
632mm

de
As As =
As ,
=
3843 mm
2

32 I

(632-222
I
[F =
0 ; Cc Ti =
ft :
600

0 85 (35) (a)(300) 3843(414) =

fo :
1101 75 MPa
&
.
.

Sts 1101 75 MPa


=

a
=
178 26 mm .
.

0 90

85-005 (35-28)
.

B
=
0 .
=
0
.

c =
112 83 . mm

0 65 TC
.
.

StepC :
Check if compression steel
yields TZ . .

C C

(22238-45)
.

t < by fi 424 62
May I
.

=
600 =

ts
.

>
..
CSWY ! fy 1000

If CS will not
yield :
6 =
0 90
.

Use T Cc + C=

& Mn =
0 .
90(1030 .
28)
Asty
=
0 .
85f : ab + As fi & Mn 927 25 =
. kN ·

Asty =
0 . 85tc B ,
cb + As[600 (=d)] -

StepQ :
Calc the nominal moment . capacity of the beam .

Mn =
Mn + Mnz ,

Mn =

G(d q) + C (b d) -

,
-

Mn 0 =
.
(d-E)
85 %cab +
Asty (d-d') -
if CS yields !
Mn =
0 85f ; Bicb
.

(d-E) + Ask (d-d') if -


CS doesn't yield !
> Mn =
0 85 (35) .
(178 24)(300) (600 .
-

1824) + 982(414) (600 65 -

Mn =
1030 28 kN .
.

m
A reinforced has a width of 250 mm and an effective depth of 625 mm. It is reinforced for compression having a steel area of
A’s=1250 mm^2 with steel covering of 62.5 mm measured from the centroid of the steel. F’c=21 MPa and fy=400 MPa.
a) Determine the depth of compression block for balanced condition
b) Determine the required tension reinforcement for balanced condition
c) What is the balanced moment capacity of the section?

250 0 003
.
0 85t'
.

62 5 Es
- Pbcri
mm

Cs P
.

L
=
+ - PRB

C
db -
Cu
Asa jobd
=

625
d-d'
d -

q
625 -

Ey fy/E T Aspfy
=
=

Solution :

a) Depth of compression block

Ab B Ob =
,

Cb =
625 -
C

0 .
003400/200 ,
000

Cb 375mm =

Ap =
318 75 .
mm

b. ) Reg tension reinforcement


.
for balanced condition

P :
Hi As :

Aseac As but

C's Ast's 1250 mm2


85fiab Tbi Ca Asbe
=
- =

318 7 .
-

C 0 =
.


Asp (400) (21) (318 75) (250) :
As 3556 + 1250
=
=
0 85
. .

,
t
As 4806 mm2
Assi =
3556 05 mm .
=

<
Tbi Asb fy
=

,
> The =
As bety for Asbe :
Alternative Solution :

A
085)
at SRB Check if As will on t
yield
=

t =
600
(375-42 5) .

: 500 MPaY

: CSWY ! p =
0 03074
.

Asp Pobd 0 03074(150)(425) 4806mm2


=
= =
.

c) Balanced Moment Capacity


0 =
0 65 .

Mnc Cs (d-d')
=

Mn Ma =
,
+ Muc
=

1250(400) (425 -
42 . 5)
Mn C ,
=
(d ) -
Muc =
281 15 kN .
·

=
0 . 85(21)(318 .
75)(250)(625 31875) -

Mn 943 57 kN
=
.
·

Mn 661 ,
=
.
32 kN ·

m #Mn =
0.
65(943 57) .

& Mn =
613 32 kN .
·

You might also like